Đến nội dung

Hình ảnh

Mỗi ngày một chút

* * * * * 12 Bình chọn

  • Please log in to reply
Chủ đề này có 297 trả lời

#261
Nguyễn Hoàng Lâm

Nguyễn Hoàng Lâm

    Sĩ quan

  • Thành viên
  • 312 Bài viết
Bài 117 : Giải phương trình:
$ \dfrac{2}{1+4^{-x^2}}+2^{-x^2-1}+2^{x^2-1}=\sqrt{2}+\dfrac{1}{\sqrt{2}}+1 $

Đôi khi ta mất niềm tin để rồi lại tin vào điều đó một cách mạnh mẽ hơn .


#262
Nguyễn Hoàng Lâm

Nguyễn Hoàng Lâm

    Sĩ quan

  • Thành viên
  • 312 Bài viết
Bài 118: Cho $ a,b,c >0 $ . Chứng minh rằng:
$ \dfrac{ab^2}{a^2+2b^2+c^2}+\dfrac{bc^2}{b^2+2c^2+a^2} +\dfrac{ca^2}{c^2+2a^2+b^2} \leq \dfrac{a+b+c}{4} $

Đôi khi ta mất niềm tin để rồi lại tin vào điều đó một cách mạnh mẽ hơn .


#263
Didier

Didier

    đẹp zai có một ko hai

  • Thành viên
  • 403 Bài viết

Bài 118: Cho $ a,b,c >0 $ . Chứng minh rằng:
$ \dfrac{ab^2}{a^2+2b^2+c^2}+\dfrac{bc^2}{b^2+2c^2+a^2} +\dfrac{ca^2}{c^2+2a^2+b^2} \leq \dfrac{a+b+c}{4} $

$\sum_{cyc}\dfrac{ab^{2}}{a^{2}+2b^{2}+c^{2 }}\leq \sum_{cyc}\dfrac{ab^{2}}{2ab+2bc}=\sum_{cyc}\dfrac{ab}{2(a+b)}$
lại có $\dfrac{ab}{a+b}\leq \dfrac{a+b}{4}$
$\Rightarrow \sum_{cyc}\dfrac{ab}{2(a+b)}\leq \dfrac{a+b+c}{4}$


#264
Ispectorgadget

Ispectorgadget

    Nothing

  • Quản lý Toán Phổ thông
  • 2946 Bài viết
Anh ơi hình như có vấn đề rồi $\frac{ab^2}{2ab+2bc}=\frac{ab^2}{2b(a+c)}=\frac{ab}{2(a+c)}$ mới đúng chứ :(

►|| The aim of life is self-development. To realize one's nature perfectly - that is what each of us is here for. ™ ♫


#265
Crystal

Crystal

    ANGRY BIRDS

  • Hiệp sỹ
  • 5534 Bài viết
Bài 119: Cho số nguyên tố $p > 3$, với mỗi $k \in \left\{ {1,2,...,p - 1} \right\}$ chọn duy nhất một số ${x_k}:\,\,k{x_k} \equiv 1\,\,\left( {\bmod p} \right)$. Các số ${n_k}$ xác định bởi $k{x_k} = 1 + p{n_k};\forall k \in \left\{ {1,2,...,p - 1} \right\}$. Chứng minh rằng:
$$\sum\limits_{k = 1}^n {k{n_k}} \equiv \frac{{p - 1}}{2}\,\,\left( {\bmod p} \right)$$

P/s: Lâm thống kê lại những bài chưa có lời giải đi.

#266
taminhhoang10a1

taminhhoang10a1

    Trung sĩ

  • Thành viên
  • 131 Bài viết
Bài 120: so sánh hai số sau: $a^b$ và $b^a$ với a,b nguyên dương
THPT THÁI NINH - THÁI THỤY - THÁI BÌNH

#267
Crystal

Crystal

    ANGRY BIRDS

  • Hiệp sỹ
  • 5534 Bài viết

Bài 120: so sánh hai số sau: $a^b$ và $b^a$ với a,b nguyên dương


Có thể dựa vào hướng sau:

Lấy logarit tự nhiên cả hai số: $a\ln b,b\ln a$. Chia cả hai số cho $ab$, ta được: $\dfrac{{\ln b}}{b}$ và $\dfrac{{\ln a}}{a}$.

Khảo sát tính đơn điệu của hàm số: $\boxed{f\left( x \right) = \dfrac{{\ln x}}{x},\,\,\forall x \in {\mathbb{Z}^ + }}$. Từ đó suy ra kết quả.

#268
Crystal

Crystal

    ANGRY BIRDS

  • Hiệp sỹ
  • 5534 Bài viết
Bài 121: Tìm số cặp khác nhau của các tập con không giao nhau thuộc tập hợp $n$ phần tử.

#269
Crystal

Crystal

    ANGRY BIRDS

  • Hiệp sỹ
  • 5534 Bài viết

Bài 115: Giải phương trình :
$ \dfrac{8}{2^{x-1}+1} + \dfrac{2^x}{2^x +1} = \dfrac{18}{2^{x-1}+2^{1-x}+2} $


bài này có vẻ chỉ đơn thuần là đặt ẩn phụ:

đặt: $ 2^{x-1}=t >0 $, PT trở thành:

$ \dfrac{8}{t+1}+\dfrac{2t}{2t+1}=\dfrac{18t}{(t+1)^2} $

$ \leftrightarrow (2t+1)(8t+8)+2t(t+1)^2=18t(2t+1) $

$ \leftrightarrow 2t^3-16t^2+8t+8=0 $

hix, nhìn cái PT cuối nghiệm xấu này nản chẳng muốn giải nữa, anh em thông cảm nhé


Cách này nhé.

Phương trình được viết thành: $$\dfrac{8}{{{2^{x - 1}} + 1}} + \dfrac{1}{{{2^{1 - x}} + 1}} = \dfrac{{18}}{{{2^{x - 1}} + {2^{1 - x}} + 2}}$$
Đặt $u = {2^{x - 1}} + 1,\,\,v = {2^{1 - x}} + 1;\,\,u,v > 0$. Ta có $uv = u + v$, do đó có hệ phương trình:
$$\left\{ \begin{array}{l}
\dfrac{8}{u} + \dfrac{1}{v} = \dfrac{{18}}{{u + v}}\\
uv = u + v
\end{array} \right. \Leftrightarrow \left[ \begin{array}{l}
\left\{ \begin{array}{l}
u = 2\\
v = 2
\end{array} \right.\\
\left\{ \begin{array}{l}
u = 9\\
v = \dfrac{9}{8}
\end{array} \right.
\end{array} \right. \Leftrightarrow \left[ \begin{array}{l}
{2^{x - 1}} + 1 = 2\\
{2^{x - 1}} + 1 = 9
\end{array} \right. \Leftrightarrow \left[ \begin{array}{l}
{2^{x - 1}} = 1\\
{2^{x - 1}} = 8
\end{array} \right. \Leftrightarrow \left[ \begin{array}{l}
x = 1\\
x = 4
\end{array} \right.$$
Nghiệm là $\boxed{x = \left\{ {1,4} \right\}}$
____________________________________________________________________
@ NGOCTIEN_A1_DQH: Nghiệm không xấu đâu, đẹp đấy chứ.

#270
Crystal

Crystal

    ANGRY BIRDS

  • Hiệp sỹ
  • 5534 Bài viết
Bài 122: (nhẹ nhàng)

Cho $x,y,z$ là các số thực dương thoả $x+y+z=1$. Tìm giá trị lớn nhất của: $$Q = \dfrac{x}{{x + yz}} + \dfrac{y}{{y + xz}} + \dfrac{{\sqrt {xyz} }}{{z + xy}}$$

#271
Crystal

Crystal

    ANGRY BIRDS

  • Hiệp sỹ
  • 5534 Bài viết

NHỮNG BÀI TOÁN CHƯA CÓ LỜI GIẢI TRONG TOPIC NÀY


Bài 2: Cho dãy số {$ x_n$ } được xác định bởi: $ \left\{\begin{array}{l}{x_0=1}\\{x_{n+1}=2+\sqrt{x_n}-2\sqrt{1+\sqrt{x_n}}} \end{array}\right. $
Ta xác đinh dãy {$ y_n$} bởi công thức $ y_n=\sum_{k=1}^{n}x_k.2^k, \forall n \in N^{*} $.Tìm công thức tổng quát của dãy {$ y_n$ } .

Bài 6: Giải phương trình nghiệm nguyên trên tập $ N ^{*} $: $ (x+y)(1+xy)=2^z $

Bài 8: Tìm các giá trị của $a,b$ sao cho :
$ P(x) = x^4-3ax^3+5x^2+b$ có nghiệm bội 2, mà nó bằng tổng của 2 nghiệm còn lại.

Bài 9: Cho $ a,b,c >0 $. Chứng minh rằng:
$$ \dfrac{a^2}{b}+\dfrac{b^2}{c}+\dfrac{c^2}{a}+4(\dfrac{bc}{b+c}+\dfrac{ca}{c+a}+\dfrac{ab}{a+b}) \geq 3(a+b+c) $$

Bài 13: Tìm 2001 số nguyên $ x_1 ,x_2 ,...,x_{2001} \in \,\overline {1...1975} $ sao cho biểu thức $$ P = \dfrac{{x_1^2 + x_2^2 + ... + x_{2001}^2 }}{{\left( {x_1 + x_2 + ... + x_{2001} } \right)^2 }}$$ đạt giá trị lớn nhất.

Bài 24: Chứng minh rằng tồn tại vô số số $n \in N^* $ sao cho
$$5x_n^{1992} + 5x_n^{1954} + 4x_n^{1975} + 8x_n^{1945} + 2x_n^{1930} + 11x_n^2 + 48\,\, \vdots \,1992$$

Bài 29: Cho lục giác đều. Chia mỗi cạnh của lục giác đều thành 1000 phần bằng nhau. Nối các điểm chia với nhau thành bởi các đoạn thẳng song song với các cạnh của lục giác. Mỗi giao điểm của các đoạn nói trên gọi là một nút. Tô màu các nút theo quy tắc sau: Mỗi lần tô 3 nút chưa có màu và là 3 đỉnh của một tam giác đều. Hỏi sau một số lần tô ta mà còn còn lại mổ nút không có màu thì đó có thể là đỉnh của lục giác hay không?

Bài 33: Giải phương trình sau trên tập các số nguyên $n \ge 2$.
$$\left[ {\sqrt n } \right] + \left[ {\sqrt[3]{n}} \right] + \left[ {\sqrt[4]{n}} \right] + ... + \left[ {\sqrt[n]{n}} \right] = \left[ {\log _2 n} \right] + \left[ {\log _3 n} \right] + ... + \left[ {\log _n n} \right]$$.

Bài 34: Gọi d là tổng độ dài các đường chéo của một đa giác lồi trong mặt phẳng có n đỉnh, n>3. Gọi p là chu vi đa giác đó.
Chứng minh rằng: $$n - 3 < \dfrac{{2d}}{p} < \left[ {\dfrac{n}{2}} \right].\left[ {\dfrac{{n + 1}}{2}} \right] - 2$$.

Bài 37: Chứng minh rằng: $$ sin e\sqrt[3]{cos (e-1)} - sin (e-1)\sqrt[3]{cos e} > \sqrt[3]{cos e . cos (e-1)} $$

Bài 43: Tìm tất cả số nguyên dương n sao cho mọi số nguyên dương với n chữ số, một trong chúng là 7 và những chữ số khác bằng 1 là một số nguyên tố.

Bài 45: Cho bát giác $A_1 A_2 ...A_8 $ nội tiếp đường tròn tâm 0. Chứng minh rằng nếu các đường chéo $A_1 A_5 ,A_2 A_6 ,A_3 A_7 ,A_4 A_8 $ đồng quy tại H (H không trùng 0) thì giao điểm của các cặp đường chéo $A_1 A_3 $ và $A_5 A_7 $, $A_1 A_7 $ và $A_3 A_5 $, $A_2 A_8 $ và $A_4 A_6 $ (nếu tồn tại) cùng nằm trên một đường thẳng.

Bài 54: Tìm tham số a để hệ sau có nghiệm:
$$\left\{ {\begin{array}{*{20}{l}}
{a{{\left( {x - a} \right)}^2}\left( {x - 2\sqrt 2 } \right) + 1 \leqslant 0} \\
{x > a > 0}
\end{array}} \right.$$

Bài 88: Cho $x,y,z > 0$ thỏa $x + y + z = 1$. Tìm GTNN của: $$P = {x^3} + \sqrt {1 + {y^2}} + \sqrt[4]{{1 + {z^4}}}$$

Bài 99: Ta gọi đoạn thẳng $\left[ {m,n} \right]$ là đoạn thẳng tốt nếu với mọi bộ ba số thực $a, b, c$ thỏa mãn điều kiện $2a + 3b + 6c = 0$ thì phương trình $a{x^2} + bx + c = 0$ có nghiệm thực thuộc đoạn $\left[ {m,n} \right]$. Trong tất cả các đoạn thẳng tốt, hãy tìm đoạn có độ dài nhỏ nhất.

Bài 107: Chứng minh đẳng thức sau: $$\sqrt[3]{{c{\rm{os}}\dfrac{{2\pi }}{7}}} + \sqrt[3]{{c{\rm{os}}\dfrac{{4\pi }}{7}}} + \sqrt[3]{{c{\rm{os}}\dfrac{{6\pi }}{7}}} = \sqrt[3]{{\dfrac{{5 - 3\sqrt[3]{7}}}{2}}}$$

Bài 114: Trong một cái bánh hình vuông cạnh $8$ cm có $32$ hạt vừng.Chứng minh rằng tồn tại hai hạt vừng mà khỏng cách tới nhau nhỏ hơn $2$ cm

Bài 116: Cho các số thực dương $a,b,c$ thỏa mãn $a+b+c=3$. Chứng minh rằng
$$ \dfrac{a}{b+c^2}+\dfrac{b}{c+a^2}+\dfrac{c}{a+b^2}+\dfrac{ab+bc+ca}{72} \ge \dfrac{37}{24} $$

Bài 117: Giải phương trình: $$ \dfrac{2}{1+4^{-x^2}}+2^{-x^2-1}+2^{x^2-1}=\sqrt{2}+\dfrac{1}{\sqrt{2}}+1 $$

Bài 119: Cho số nguyên tố $p > 3$, với mỗi $k \in \left\{ {1,2,...,p - 1} \right\}$ chọn duy nhất một số ${x_k}:\,\,k{x_k} \equiv 1\,\,\left( {\bmod p} \right)$. Các số ${n_k}$ xác định bởi $k{x_k} = 1 + p{n_k};\forall k \in \left\{ {1,2,...,p - 1} \right\}$. Chứng minh rằng:
$$\sum\limits_{k = 1}^n {k{n_k}} \equiv \dfrac{{p - 1}}{2}\,\,\left( {\bmod p} \right)$$

Bài 121: Tìm số cặp khác nhau của các tập con không giao nhau thuộc tập hợp $n$ phần tử.

Bài 122: Cho $x,y,z$ là các số thực dương thoả $x+y+z=1$. Tìm giá trị lớn nhất của: $$Q = \dfrac{x}{{x + yz}} + \dfrac{y}{{y + xz}} + \dfrac{{\sqrt {xyz} }}{{z + xy}}$$

Bài viết đã được chỉnh sửa nội dung bởi xusinst: 08-01-2012 - 11:38


#272
Crystal

Crystal

    ANGRY BIRDS

  • Hiệp sỹ
  • 5534 Bài viết
Chào các bạn! Trước hết, mình xin thay mặt chủ Topic này là Lâm, kêu gọi sự tham gia, ủng hộ của mọi người. Để đáp lại lời kêu gọi này, mọi người hãy cùng giải quyết những bài toán trên.

Đừng quên là xong khi giải xong một bài (hay nhiều hơn) thì bạn hãy góp cho pic này một bài nhé. Cảm ơn!

#273
E. Galois

E. Galois

    Chú lùn thứ 8

  • Quản lý Toán Phổ thông
  • 3861 Bài viết

Cảm ơn Thành vì tổng hợp rất cần thiết ở trên.

Bài 61:

Giả sử f(0)=0 và f có đạo hàm tại điểm 0. Tính

$$L=\mathop {\lim }\limits_{x \to 0} \dfrac{1}{x}\left[ {f\left( x \right) + f\left( {\dfrac{x}{2}} \right) + f\left( {\dfrac{x}{3}} \right) + ... + f\left( {\dfrac{x}{k}} \right)} \right]\,\,,\,\,k \in {Z^ + }$$.


$ k \in \mathbb{N}^*$ chứ nhỉ?

$\forall k \in \mathbb{N}^*$, ta có:
$$L=\lim_{x \to 0} \sum_{i=1}^k\dfrac{f\left ( \dfrac{x}{i} \right )}{x}=\sum_{i=1}^k\lim_{x \to 0} \dfrac{f\left ( \dfrac{x}{i} \right )}{x} $$
Mặt khác, vì $f(x)$ có đạo hàm tại 0 nên
$$\lim_{x \to 0} \dfrac{f\left ( \dfrac{x}{i} \right )}{x} = \dfrac{1}{i}\lim_{\dfrac{x}{i} \to 0} \dfrac{f\left ( 0 +\dfrac{x}{i} \right )-f(0)}{\dfrac{x}{i}} = \dfrac{1}{i}.f'(0), \forall i \in \mathbb{N}^*$$
Do đó:
$$L=f'(0).\sum_{i=1}^k \dfrac{1}{i}$$


1) Xem cách đăng bài tại đây
2) Học gõ công thức toán tại: http://diendantoanho...oạn-thảo-latex/
3) Xin đừng đặt tiêu đề gây nhiễu: "Một bài hay", "... đây", "giúp tớ với", "cần gấp", ...
4) Ghé thăm tôi tại 
http://Chúlùnthứ8.vn

5) Xin đừng hỏi bài hay nhờ tôi giải toán. Tôi cực gà.


#274
Crystal

Crystal

    ANGRY BIRDS

  • Hiệp sỹ
  • 5534 Bài viết

$ k \in \mathbb{N}^*$ chứ nhỉ?


Thay mặt Lâm, em cảm ơn anh Thế đã nổ phát súng đầu tiên sau "lời kêu gọi toàn quốc kháng chiến" :D.
______________________________________
Trở lại thắc mắc của anh. Em nghĩ $k\in \mathbb{N}^{*}$ hay $k\in \mathbb{Z}^{+}$ thì nó không ảnh hưởng đến bài toán. Vì trong tập số nguyên thì có thể xem tập số tự nhiên khác không với tập số nguyên dương là tương đương.

Ý kiến của em như thế có đúng không? Anh cho em ý kiến nhé.

#275
E. Galois

E. Galois

    Chú lùn thứ 8

  • Quản lý Toán Phổ thông
  • 3861 Bài viết

Thay mặt Lâm, em cảm ơn anh Thế đã nổ phát súng đầu tiên sau "lời kêu gọi toàn quốc kháng chiến" :D.
______________________________________
Trở lại thắc mắc của anh. Em nghĩ $k\in \mathbb{N}^{*}$ hay $k\in \mathbb{Z}^{+}$ thì nó không ảnh hưởng đến bài toán. Vì trong tập số nguyên thì có thể xem tập số tự nhiên khác không với tập số nguyên dương là tương đương.

Ý kiến của em như thế có đúng không? Anh cho em ý kiến nhé.


Kí hiệu $\mathbb{Z}_+$ là để chỉ tập các số nguyên không âm chứ nhỉ? Có nghiã là tập này có chứa số 0.

Thành còn nhớ công thức tính tổng $\sum_{i=1}^k \dfrac{1}{i}$ không?

1) Xem cách đăng bài tại đây
2) Học gõ công thức toán tại: http://diendantoanho...oạn-thảo-latex/
3) Xin đừng đặt tiêu đề gây nhiễu: "Một bài hay", "... đây", "giúp tớ với", "cần gấp", ...
4) Ghé thăm tôi tại 
http://Chúlùnthứ8.vn

5) Xin đừng hỏi bài hay nhờ tôi giải toán. Tôi cực gà.


#276
Crystal

Crystal

    ANGRY BIRDS

  • Hiệp sỹ
  • 5534 Bài viết
Dạ vâng! Cảm ơn anh đã nhắc nhở. Em nhầm kí hiệu :D.
______________________________________

Bài trên ra kết quả như thế là được rồi anh à, không cần tính tổng $\sum_{i=1}^k \dfrac{1}{i}$ đâu.

Bài viết đã được chỉnh sửa nội dung bởi xusinst: 18-12-2011 - 00:16


#277
E. Galois

E. Galois

    Chú lùn thứ 8

  • Quản lý Toán Phổ thông
  • 3861 Bài viết

Bài 46:

Tìm số các hoán vị không có điểm cố định của tập hợp $A = \left\{ {1,2,...,n} \right\}$.


Do mình chưa hiểu đề nên mình sẽ giải bài này theo cả 2 cách hiểu.

Cách hiểu 1: không có điểm cố định tức là các hoán vị của tập hợp $A$ kể trên không có bất kì vị trí nào giống nhau. Ta có $n$ hoán vị như vậy:
$$(1,2,...,n);(2,3,...,n,1);(3,4,...,n,1,2); ...; (n,1,2, ..., n-1)$$

Cách hiểu 2:Gọi hoán vị $(1,2,3,...,n)$ là hoán vị chuẩn. Không có điểm cố định tức là các hoán vị còn lại không có vị trí nào giống với hoán vị chuẩn.
Ta có 1 hoán vị chuẩn: $(1,2,3,...,n)$
Để xây dựng một hoán vị khác với hoán vị chuẩn và không có điểm cố định, ta làm như sau:
- Chọn vị trí cho số $1$. Vì hoán vị đang xây dựng không có điểm cố định nên $1$ không được đứng ở đầu, vậy ta có $n-1$ vị trí để đặt số $1$.
Không giảm tổng quát, ta giả sử rằng, số $1$ được đặt vào vị trí của số $2$.
- Chọn vị trí cho số $2$. Vì $1$ đã được đặt vào vị trí của $2$ nên ta có $n-1$ cách đặt $2$ vào các vị trí còn lại để không trùng với vị trí của $2$ trong hoán vị chuẩn.

Không giảm tổng quát, ta giả sử rằng, số $2$ được đặt vào vị trí của số $3$.
- Chọn vị trí cho số $3$. Vì $1$ và $2$ đã được đặt vào 2 vị trí của nên ta có $n-2$ cách đặt $3$ vào các vị trí còn lại để không trùng với vị trí của $3$ trong hoán vị chuẩn.
...
tương tự cho các số từ $4$ đến $n-2$
- Dễ thấy $n-2$ có 3 vị trí để đặt. Không giảm tổng quát, ta giả sử rằng, số $n-2$ được đặt vào vị trí của số $n-1$.
- Ta còn 2 số là $n-1$ và $n$ và còn 2 vị trí để đặt. Nhưng nếu đặt $n$ vào vào vị trí cuối cùng thì ta sẽ được 1 hoán vị có vị trí cuối trùng hoán vị chuẩn. Vậy chỉ có cách xếp duy nhất là đặt $n-1$ vào vị trí của $n$, còn $n$ thì xếp vào vị trí trống còn lại.

Vậy ta có số hoán vị cần tìm là: $1 + (n-1)\dfrac{(n-1)!}{2}$

1) Xem cách đăng bài tại đây
2) Học gõ công thức toán tại: http://diendantoanho...oạn-thảo-latex/
3) Xin đừng đặt tiêu đề gây nhiễu: "Một bài hay", "... đây", "giúp tớ với", "cần gấp", ...
4) Ghé thăm tôi tại 
http://Chúlùnthứ8.vn

5) Xin đừng hỏi bài hay nhờ tôi giải toán. Tôi cực gà.


#278
hxthanh

hxthanh

    Tín đồ $\sum$

  • Hiệp sỹ
  • 3921 Bài viết
Bài toán được hiểu theo nghĩa 2, Galois kiểm tra kết quả lại nhé, thử với $n=2$ xem!

Kết quả phải là:
$$S_n=n!\sum_{k=0}^n\dfrac{(-1)^k}{k!}=\left\lfloor\dfrac{n!+1}{e}\right\rfloor$$
_______________________
from here


E.Galois: Hic! Xấu hổ quá!

#279
Crystal

Crystal

    ANGRY BIRDS

  • Hiệp sỹ
  • 5534 Bài viết
Dạ đúng rồi ạ. Bài toán được hiểu theo nghĩa 2 và đáp án thầy Thanh đưa ra là hoàn toàn chính xác.

#280
E. Galois

E. Galois

    Chú lùn thứ 8

  • Quản lý Toán Phổ thông
  • 3861 Bài viết
Chán quá, mình đi tự tử đây!
Xấu hổ chết đi được!

1) Xem cách đăng bài tại đây
2) Học gõ công thức toán tại: http://diendantoanho...oạn-thảo-latex/
3) Xin đừng đặt tiêu đề gây nhiễu: "Một bài hay", "... đây", "giúp tớ với", "cần gấp", ...
4) Ghé thăm tôi tại 
http://Chúlùnthứ8.vn

5) Xin đừng hỏi bài hay nhờ tôi giải toán. Tôi cực gà.





0 người đang xem chủ đề

0 thành viên, 0 khách, 0 thành viên ẩn danh